Find the curvature at the point (x, y) on the ellipse?

Click For Summary
To find the curvature at a point on the ellipse defined by the equation x^2/9 + y^2/4 = 1, the parameters a and b are identified as 3 and 2, respectively. The parametric equations x(t) = 3cos(t) and y(t) = 2sin(t) are provided, but it's noted that converting to parametric form is not necessary for calculating curvature. The curvature formula is given as 162/(81 - 5x^2)^(3/2). It is suggested to refer to the textbook for curvature formulas, as they are typically included in related problems. Understanding the curvature concept is essential for solving the problem effectively.
Math10
Messages
301
Reaction score
0

Homework Statement


Find the curvature at the point (x, y) on the ellipse x^2/9+y^2/4=1.

Homework Equations


None.

The Attempt at a Solution


x^2/a^2+y^2/b^2=1
so I know that a=3 and b=2 for this problem.
x(t)=acos(t) and y(t)=bsin(t)
so x(t)=3cos(t) and y(t)=2sin(t)
now what? What's the formula for finding the curvature for this problem? The answer is 162/(81-5x^2)^(3/2).
 
Physics news on Phys.org
How did you define curvature?
 
Math10 said:

Homework Statement


Find the curvature at the point (x, y) on the ellipse x^2/9+y^2/4=1.

Homework Equations


None.

The Attempt at a Solution


x^2/a^2+y^2/b^2=1
so I know that a=3 and b=2 for this problem.
x(t)=acos(t) and y(t)=bsin(t)
so x(t)=3cos(t) and y(t)=2sin(t)
now what? What's the formula for finding the curvature for this problem? The answer is 162/(81-5x^2)^(3/2).
If you textbook has a problem about finding curvature, I can pretty much guaranteed that it will have a formula for curvature somewhere close by in the book. That should be the first place you look.

Also, you don't need to convert to parametric form to find the curvature.
 
Question: A clock's minute hand has length 4 and its hour hand has length 3. What is the distance between the tips at the moment when it is increasing most rapidly?(Putnam Exam Question) Answer: Making assumption that both the hands moves at constant angular velocities, the answer is ## \sqrt{7} .## But don't you think this assumption is somewhat doubtful and wrong?

Similar threads

  • · Replies 4 ·
Replies
4
Views
3K
  • · Replies 11 ·
Replies
11
Views
3K
  • · Replies 1 ·
Replies
1
Views
2K
  • · Replies 5 ·
Replies
5
Views
2K
  • · Replies 1 ·
Replies
1
Views
2K
Replies
3
Views
2K
Replies
8
Views
2K
Replies
12
Views
2K
  • · Replies 5 ·
Replies
5
Views
2K
  • · Replies 10 ·
Replies
10
Views
2K